Jump to content

tpdtthltvp's Content

There have been 806 items by tpdtthltvp (Search limited from 08-06-2020)



Sort by                Order  

#647214 Cho x, y, z>0 : CMR: $\sum \frac{x^{3}...

Posted by tpdtthltvp on 30-07-2016 - 18:08 in Bất đẳng thức và cực trị

Vậy còn hai số 3 và 8 này em chọn như thế nào?

 Ta đi chứng minh $\sum \frac{1}{(1+a)^3}\geq m.(\sum \frac{1}{a^{2k}+a^k+1})$ mà dấu $"="$ khi $a=1$ và $\sum \frac{1}{a^{2k}+a^k+1}\geq 1$ nên từ đó chọn được $m$ thích hợp.

 Cũng có cách "cùi" hơn là thấy đề yêu cầu chứng minh $\sum \frac{1}{(1+a)^3}\geq \frac{3}{8}$ nên chọn ngay $\frac{3}{8}$ :D




#647082 Cho x, y, z>0 : CMR: $\sum \frac{x^{3}...

Posted by tpdtthltvp on 29-07-2016 - 20:19 in Bất đẳng thức và cực trị

Lời giải của Khánh rất hay. Anh không biết sao mà em tìm được bđt trung gian như vậy

 

À với cách đặt như vậy thì $abc=1$ nên em liên tưởng tới BĐT $vasc: \;\;\;\;\;\;\;\ \sum \frac{1}{a^{2k}+a^{k}+1}\geq 1.$

Ta sẽ chứng minh: $\frac{1}{(1+a)^3}\geq \frac{3}{8(a^{2k}+a^k+1)}\Leftrightarrow 8a^{2k}+8a^k+5\geq 3a^3+9a^2+9a$

Để có được như vậy thì đẳng thức phải xảy ra, tức là $8a^{2k}+8a^k+5= 3a^3+9a^2+9a$

Đạo hàm cả $2$ vế thì được $16k.a^{2k-1}+8k.a^{k-1}=9a^2+18a+9$

Mà dấu $"="$ xảy ra khi $x=y=z$ nên $a=1,$ từ đó tính được $k=\frac{3}{2}.$

Việc cuối cùng chỉ là đi chứng minh BĐT: $\frac{1}{(1+a)^3}\geq \frac{3}{8(a^3+\sqrt{a^3}+1)}$ là đúng.

---------------------------------------

Em cũng không biết tại sao anh lại nghĩ đến chứng minh BĐT: $(1+a)^3 \leq (1+abc)(1+\frac{a}{b})(1+\frac{a}{c})$




#647058 Cho x, y, z>0 : CMR: $\sum \frac{x^{3}...

Posted by tpdtthltvp on 29-07-2016 - 17:52 in Bất đẳng thức và cực trị

Cho x, y, z>0 :

CMR: $\frac{x^{3}}{(x+y)^{3}}+\frac{y^{3}}{(y+z)^{3}}+\frac{z^{3}}{(z+x)^{3}}\geq \frac{3}{8}$

Lời giải:

Ta có bất đẳng thức cần chứng minh tương đương với:

$$\frac{1}{(1+\frac{y}{x})^3}+\frac{1}{(1+\frac{z}{y})^3}+\frac{1}{(1+\frac{x}{z})^3}\geq \frac{3}{8}\;\;\;\;\;\;\;\;\;\;\;\;\;\;\;\;\;\;\;\;\ (1)$$

Đặt $a=\frac{y}{x};b=\frac{z}{y};c=\frac{x}{z}$ thì $abc=1$ và $(1)$ viết lại thành:

$$\frac{1}{(1+a)^3}+\frac{1}{(1+b)^3}+\frac{1}{(1+c)^3}\geq \frac{3}{8}$$

Ta sẽ chứng minh:

$$\frac{1}{(1+a)^3}\geq \frac{3}{8(a^3+\sqrt{a^3}+1)}$$

$$\Leftrightarrow 8a^3+8\sqrt{a^3}+8\geq 3a^3+9a^2+9a+3$$

$$\Leftrightarrow 5a^3+8\sqrt{a^3}+5\geq 9a^2+9a\;\;\;\;\;\;\;\;\;\;\ (2)$$

Bất đẳng thức trên luôn đúng theo $\textrm{AM-GM}:$

  • $$3a^3+6\sqrt{a^3}=3(a^3+\sqrt{a^3}+\sqrt{a^3})\geq 3.3a^2=9a^2$$
  • $$2a^3+2\sqrt{a^3}+5=a^3+a^3+\sqrt{a^3}+\sqrt{a^3}+1+1+1+1+1\geq 9a$$

Cộng $(2)$ vế BĐT trên lại ta được $(2)$ đúng.

Chứng minh tương tự với $b,c$ suy ra:

$$\frac{1}{(1+a)^3}+\frac{1}{(1+b)^3}+\frac{1}{(1+c)^3}\geq \frac{3}{8(a^3+\sqrt{a^3}+1)}+\frac{3}{8(b^3+\sqrt{b^3}+1)}+\frac{3}{8(c^3+\sqrt{c^3}+1)}$$

Cần chứng minh:

$$\frac{1}{a^3+\sqrt{a^3}+1}+\frac{1}{b^3+\sqrt{b^3}+1}+\frac{1}{c^3+\sqrt{c^3}+1}\geq 1$$

Nếu đặt $\sqrt{a^3}=m; \sqrt{b^3}=n; \sqrt{c^3}=p$ thì BĐT này trở nên quen thuộc:

$$\frac{1}{m^2+m+1}+\frac{1}{n^2+n+1}+\frac{1}{p^2+p+1}\geq 1.$$

Chứng minh

Vậy ta có đpcm.

 

P/S




#646938 Marathon Phương trình và hệ phương trình VMF

Posted by tpdtthltvp on 28-07-2016 - 20:44 in Phương trình - hệ phương trình - bất phương trình

Bài 92: Giải hệ phương trình:

$\left\{\begin{matrix} x_1+x_2+x_3+x_4+x_5=1\\ (x_1+x_2)(x_1+x_2+x_3)(x_1+x_2+x_3+x_4)=256x_1x_2x_3x_4x_5\\ x_1,x_2,x_3,x_4,x_5> 0\\ \end{matrix}\right.$

Bài 92:  Sử dụng BĐT quen thuộc $(a+b)^2 \geq 4ab,$ ta có:
$(x_1+x_2)(x_1+x_2+x_3)(x_1+x_2+x_3+x_4)=(x_1+x_2+x_3+x_4+x_5)^2(x_1+x_2+x_3+x_4)(x_1+x_2+x_3)(x_1+x_2)\geq 4(x_1+x_2+x_3+x_4)^2(x_1+x_2+x_3)(x_1+x_2)x_5\geq 16(x_1+x_2+x_3)^2(x_1+x_2)x_4x_5\geq 64(x_1+x_2)^2x_3x_4x_5\geq 256x_1x_2x_3x_4x_5$
Dấu đẳng thức xảy ra khi:
$\left\{\begin{matrix} x_5=x_4+x_3+x_2+x_1 \\ x_4=x_3+x_2+x_1 \\ x_3=x_2+x_1 \\ x_2=x_1 \\ x_1+x_2+x_3+x_4+x_5=1 \end{matrix}\right.\Leftrightarrow x_5=\frac{1}{2};x_4=\frac{1}{4};x_3=\frac{1}{8};x_2=x_1=\frac{1}{16}$



#646791 [Đại số] THCS tháng 11: Bài toán về điểm nguyên trên mặt phẳng toạ độ

Posted by tpdtthltvp on 27-07-2016 - 20:39 in Thảo luận đề thi VMEO IV

Mặc dù hết hạn rồi nhưng em mới học nên cũng thử sức làm phần $(1),$ đúng sai mọi người cứ ném đá.  :ph34r:

Bài toán có thể phát biểu lại: Mọi đường thẳng $y=x+n$ với $\frac{1}{\sqrt{2}}-\frac{1}{8}\leq n\leq \frac{1}{\sqrt{2}}+\frac{1}{8}$ không đi qua bất kì iểm nguyên nào hay phương trình $y=x+n$ không có nghiệm nguyên.

Giả sử phương trình có nghiệm nguyên $(x_0,y_0$) thì $x_0,y_0\in \mathbb{Z}$ và $y_0=x_0+n$ suy ra $n\in \mathbb{Z}$ mà $\frac{1}{\sqrt{2}}-\frac{1}{8}\leq n\leq \frac{1}{\sqrt{2}}+\frac{1}{8}$ nên $n\notin \mathbb{Z}.$

Vậy ta có đpcm.




#646036 Thông tin về VMF trên Alexa

Posted by tpdtthltvp on 22-07-2016 - 19:26 in Diễn đàn Toán học trên chặng đường phát triển

Vậy là diễn đàn bị tụt hạng rồi :(

alexa1.PNG




#645313 $(4 +\sqrt{15})(\sqrt{10}-\sqrt{6})\sqrt{4-\sqr...

Posted by tpdtthltvp on 17-07-2016 - 18:26 in Đại số

 

Cách bạn hơi "tay chân" :D

 

 

$(4 +\sqrt{15})(\sqrt{10}-\sqrt{6})\sqrt{4-\sqrt{15}}$

$(4 +\sqrt{15})(\sqrt{10}-\sqrt{6})\sqrt{4-\sqrt{15}}=\sqrt{2}(4+\sqrt{15})(\sqrt{5}-\sqrt{3}).\frac{\sqrt{5}-\sqrt{3}}{\sqrt{2}}=(4+\sqrt{15})(\sqrt{5}-\sqrt{3})^2=2(4+\sqrt{15})(4-\sqrt{15})=2$




#645194 Hỏi sau 95 lần thực hiện số còn lại trên bảng là số nào?

Posted by tpdtthltvp on 16-07-2016 - 19:35 in Toán rời rạc

Trên bảng có các số 1/96; 2/96; ....; 96/96. Mỗi lần thực hiện cho phép xoá đi 2 số a,b và thay bằng số mới bằng a+b-2ab. Hỏi sau 95 lần thực hiện số còn lại trên bảng là số nào?

Nhận thấy $a+b-2ab=(a-\frac{1}{2})(1-2b)+\frac{1}{2}$ nên khi thực hiện như vậy với một số bất kì và $\frac{48}{96}(=\frac{1}{2}),$ ta luôn được kết quả là $\frac{1}{2}$

Cuối cùng ta thu được kết quả là $\frac{1}{2}$




#645130 $\sum \frac{1}{a^{5}+b^{2}+...

Posted by tpdtthltvp on 16-07-2016 - 09:40 in Bất đẳng thức và cực trị

Cho abc$\geq 1$    Chứng Minh : $\sum \frac{1}{a^{5}+b^{2}+c^{2}} \leq \frac{3}{a^{2}+b^{2}+c^{2}}$

Áp dụng BĐT $Cauchy-schwwarz:$

$$(a^5+b^2+c^2)(\frac{1}{a}+b^2+c^2)\geq (a^2+b^2+c^2)$$

Suy ra:$$\sum \frac{1}{a^5+b^2+c^2}\leq \sum \frac{\frac{1}{a}+b^2+c^2}{(a^2+b^2+c^2)^2}$$

Ta chỉ cần chứng minh:

$$\sum (\frac{1}{a}+b^2+c^2)\leq 3(a^2+b^2+c^2)\Leftrightarrow \frac{1}{a}+\frac{1}{b}+\frac{1}{c}\leq a^2+b^2+c^2$$

Kết hợp với $1\leq abc$ ta được bất đẳng thức cuối đúng.

Suy ra đpcm.




#645129 CMR: $(a^{2004}-1) \vdots( b+1)$

Posted by tpdtthltvp on 16-07-2016 - 09:28 in Đại số

1) Cho $a,b\in \mathbb{Z}$, $a,b\neq -1$ thỏa mãn : $\frac{a^{2}-1}{b+1} + \frac{b^{2}-1}{a+1} \in \mathbb{Z}$

CMR: $(a^{2004}-1) \vdots( b+1)$

2) Xét 100 số tự nhiên $a_{1};a_{2};a_{3};...;a_{100}$  thỏa mãn : $\frac{1}{\sqrt{a_{1}}}+\frac{1}{\sqrt{a_{2}}}+\frac{1}{\sqrt{a_{3}}}+...+\frac{1}{\sqrt{a_{100}}}=20$

CMR trong 100 số đó có ít nhất 2 số bằng nhau

1) 

Bổ đề: Với $p,q$ là các số hữu tỉ sao cho $p+q$ và $pq$ là các số nguyên thì $p,q$ là các số nguyên.

Chứng minh

Áp dụng bổ đề (Có $\frac{a^2-1}{b+1}.\frac{b^2-1}{a+1}=(a-1)(b-1)\in \mathbb{Z}$) ta được: $\frac{a^2-1}{b+1}\in \mathbb{Z}\Leftrightarrow a^2-1\vdots b+1$ mà $a^{2004}-1\vdots a^2-1$.

Từ đó suy ra $\text{đpcm}$

 

2)

Giả sử trong $100$ số đó không có $2$ số nào bằng nhau. Thế thì: $$M=\frac{1}{\sqrt{a_{1}}}+\frac{1}{\sqrt{a_{2}}}+\frac{1}{\sqrt{a_{3}}}+...+\frac{1}{\sqrt{a_{100}}}\leq 1+\frac{1}{\sqrt{2}}+\frac{1}{\sqrt{3}}+\cdots +\frac{1}{\sqrt{100}}=N$$

Ta sẽ chứng minh $N<20.$ Thật vậy:

$$N=1+\frac{2}{2\sqrt{2}}+\frac{2}{2\sqrt{3}}+\cdots +\frac{2}{2\sqrt{100}}< 1+\frac{2}{\sqrt{1}+\sqrt{2}}+\frac{2}{\sqrt{2}+\sqrt{3}}+\cdots +\frac{2}{\sqrt{99}+\sqrt{100}}=1+2(\sqrt{2}-\sqrt{1}+\sqrt{3}-\sqrt{2}+\cdots +\sqrt{100}-\sqrt{99})=1+2(\sqrt{100}-\sqrt{1})=19<20.$$

Vậy ta có $\text{đpcm}$




#643931 UEFA EURO 2016

Posted by tpdtthltvp on 07-07-2016 - 08:14 in Góc giao lưu

Bale lực bất tòng tâm, câu chuyện cổ tích của xứ Wales tại EURO 2016 đã kết thúc.

Không biết Đức hay Pháp sẽ đối đầu với Bồ Đào Nha ở chung kết :)




#643769 Tìm Min : $\sum \frac{a}{\sqrt{a+b...

Posted by tpdtthltvp on 05-07-2016 - 20:02 in Bất đẳng thức và cực trị

Cho $a,b,c >0$ . ... $ab + bc + ca \geq 3$ .... Tìm Min : $\sum \frac{a}{\sqrt{a+b}}$

Đặt $M=\sum \frac{a}{\sqrt{a+b}},N=a(a+b)+b(b+c)+c(c+a)$

Áp dụng BĐT $Holder:$

$$M^2.N\geq (a+b+c)^3\Leftrightarrow M^2\geq \frac{(a+b+c)^3}{N}$$

Ta sẽ chứng minh:
$$\frac{(a+b+c)^3}{N}\geq \frac{9}{2}\Leftrightarrow 2(a+b+c)^3\geq 9\left [ a(a+b)+b(b+c)+c(c+a) \right ]\Leftrightarrow 2(a+b+c)^3+9(ab+bc+ca)\geq 9(a+b+c)^2$$

BĐT này đúng do:

$$2(a+b+c)^3+9(ab+bc+ca)\geq (a+b+c)^3+(a+b+c)^3+27\geq 9(a+b+c)^2 \text{ (AM-GM)}$$

Vậy $\min \sum \frac{a}{\sqrt{a+b}}=\frac{3}{\sqrt{2}}$. Đẳng thức xảy ra khi $a=b=c=1.\blacksquare $




#643557 Chứng minh $3$ điểm $C,E,N$ thẳng hàng.

Posted by tpdtthltvp on 04-07-2016 - 08:16 in Hình học

  Cho đường tròn $(O;R)$ đường kính $AB.$ Qua $B$ kẻ tiếp tuyến $d$ của đường tròn $(O).MN$ là một đường kính thay đổi của đường tròn $(M$ không trùng với $A,B).$ Các đường thẳng $AM$ và $AN$ cắt đường thẳng $d$ lần lượt tại $C$ và $D.$ Gọi $I$ là giao điểm của $CO$ và $BM.$ Đường thẳng $AI$ cắt $(O)$ tại điểm thứ hai là $E,$ cắt đường thẳng $d$ tại $F.$ Chứng minh $3$ điểm $C,E,N$ thẳng hàng.

 

hungvuong.png




#643351 Tổng hợp các bài BĐT

Posted by tpdtthltvp on 02-07-2016 - 21:29 in Bất đẳng thức và cực trị

Cho $a,b\in N\ast$ thoả $(\frac{a+1}{a}+\frac{b+1}{b})\in N\ast$. Gọi d là ước chung của a và b. CMR : $d\leq \sqrt{a+b}$

Từ $\frac{a+1}{a}+\frac{b+1}{b}\in \mathbb{N}^*$ suy ra $a+b\vdots ab$, dẫn đến $a+b\geq ab$.

Mặt khác do $d$ là ước chung của $a,b$ nên $ab\geq d^2$.

Do đó: $a+b\geq d^2\Rightarrow \sqrt{a+b}\geq d(\text{đpcm})$

 

 

Cho $a,b,c>0$ thoả $a+b+c=1$. Tìm GTNN của $F=14(a^{2}+b^{2}+c^{2})+\frac{ab+bc+ca}{a^{2}b+b^{2}c+c^{2}a}$

Ta có: $a^2+b^2+c^2=(a+b+c)(a^2+b^2+c^2)=(a^3+a^2b+ab^2)+(b^3+b^2c+bc^2)+(c^3+c^2a+ca^2)\geq 3a^2b+3b^2c+3c^2a\text{ (}AM-GM)\Rightarrow a^2b+b^2c+c^2a\leq \frac{1}{3}(a^2+b^2+c^2)$

Do đó:

$$A=14(a^2+b^2+c^2)+\frac{\frac{1-(a^2+b^2+c^2)}{2}}{a^2b+b^2c+c^2a}\geq 14(a^2+b^2+c^2)+\frac{1-(a^2+b^2+c^2)}{\frac{2}{3}(a^2+b^2+c^2)}=14(a^2+b^2+c^2)+\frac{3}{2(a^2+b^2+c^2)}-\frac{3}{2}=[\frac{27(a^2+b^2+c^2)}{2}+\frac{3}{2(a^2+b^2+c^2)}]+\frac{a^2+b^2+c^2}{2}-\frac{3}{2}\geq 9+\frac{1}{6}-\frac{3}{2}=\frac{23}{2}$$

Đẳng thức xảy ra khi $a=b=c=\frac{1}{3}.$

 

 

Cho $x,y,z>0$ thoả $xy+yz+zx=1$. CMR : $\frac{x}{\sqrt{1+x^{2}}}+\frac{y}{\sqrt{1+y^{2}}}+\frac{z}{\sqrt{1+z^{2}}}\leq \frac{3}{2}$

Có: $$\sum \frac{x}{\sqrt{1+x^2}}=\sum \frac{x}{\sqrt{xy+yz+zx+x^2}}=\sum \frac{x}{\sqrt{(x+y)(z+x)}}\leq \sum \frac{1}{2}(\frac{x}{x+y}+\frac{x}{z+x})=\frac{3}{2}$$

Đẳng thức xảy ra khi $x=y=z=\frac{\sqrt{3}}{3}$




#643014 MIN: $P=\frac{a^2}{b+2c}+\frac{b^2...

Posted by tpdtthltvp on 30-06-2016 - 21:16 in Bất đẳng thức và cực trị

Cho a,b,c dương thỏa mãn: $a^2+b^2+c^2=3$.

Tìm GTNN của: $P=\frac{a^2}{b+2c}+\frac{b^2}{c+2a}+\frac{c^2}{a+2b}$

 

 

Áp dụng BĐT $Schwarz,$ ta có:

$$\sum \frac{a^2}{b+2c}=\sum \frac{a^4}{a^2b+2a^2c}\geq \frac{(a^2+b^2+c^2)^2}{\sum a^2b+2\sum a^2c}=\frac{9}{\sum a^2b+2\sum a^2c}$$

Do đó chỉ cần chứng minh:

$$\sum a^2b+2\sum a^2c\leq 9$$

Thật vậy, BĐT trên đúng do:

  • $a^2b+b^2c+c^2a\leq \sqrt{(a^2+b^2+c^2)(a^2b^2+b^2c^2+c^2a^2)}=\sqrt{3(a^2b^2+b^2c^2+c^2a^2)}\leq \sqrt{3.\frac{(a^2+b^2+c^2)^2}{3}}=3$
  • $2(a^2c+b^2a+c^2b)\leq 2\sqrt{(a^2c^2+a^2b^2+b^2c^2)(a^2+b^2+c^2)}\leq 2\sqrt{\frac{(a^2+b^2+c^2)^2}{3}.3}=6$

Do đó BĐT được chứng minh. Dấu đẳng thức xảy ra khi $a=b=c=1$

 



#642426 MIN: $P=(a^2+b^2+c^2)(\frac{1}{a^2}+\frac...

Posted by tpdtthltvp on 27-06-2016 - 12:04 in Bất đẳng thức và cực trị

Cho a,b,c là độ dài một tam giác không nhọn.

Tìm MIN của: $P=(a^2+b^2+c^2)(\frac{1}{a^2}+\frac{1}{b^2}+\frac{1}{c^2})$

Vì $a,b,c$ là $3$ độ dài $3$ cạnh của một tam giác không nhọn nên không mất tính tổng quát, ta có thể giả sử $\widehat{A}\geq 90^{\circ}$. khi đó $a^2\geq b^2+c^2$.

Ta có:

$$P=3+\frac{b^2+c^2}{a^2}+\frac{a^2+c^2}{b^2}+\frac{a^2+b^2}{c^2}=3+(\frac{b^2+c^2}{a^2}+\frac{a^2}{b^2}+\frac{a^2}{c^2})+(\frac{b^2}{c^2}+\frac{c^2}{b^2})\geq 3+(\frac{b^2+c^2}{a^2}+\frac{4a^2}{b^2+c^2})+2=5+(\frac{b^2+c^2}{a^2}+\frac{a^2}{b^2+c^2})+\frac{3a^2}{b^2+c^2}\geq 5+2+3=10$$

Vậy $\min P=10.$ Dấu $"="$ xảy ra khi $a,b,c$ là độ dài một tam giác vuông.




#642337 Giải HPT: $\left\{\begin{matrix} \sqr...

Posted by tpdtthltvp on 26-06-2016 - 21:12 in Phương trình, hệ phương trình và bất phương trình

Giải hệ phương trình:

$$\left\{\begin{matrix} \sqrt{x+y}+\sqrt[3]{x+y+7}=3 \\ \sqrt{x^2+xy+4}+\sqrt{y^2+xy+4}=3 \end{matrix}\right.$$




#642092 $x^5+y^5+1=(x+2)^5+(y-3)^5$

Posted by tpdtthltvp on 25-06-2016 - 09:54 in Số học

Giải phương trình nghiệm nguyên: $x^5+y^5+1=(x+2)^5+(y-3)^5$.

Theo tam giác $Pascal$ ,ta có: $(x+2)^5+(y-3)^5\equiv x^5+2^5+y^5-3^5=x^5+y^5-211 (\mod 5)$

Do đó: $x^5+y^5+1\equiv x^5+y^5-211(\mod5)\Leftrightarrow 212\equiv 0(\mod5)(\text{Vô lí})$

Vậy $PT$ vô nghiệm.

 

P/S: Có lẽ đề sai. 




#641991 Chứng minh rằng: $a^2+b^2+c^2+9abc \ge 2(ab+bc+ac).$

Posted by tpdtthltvp on 24-06-2016 - 11:12 in Bất đẳng thức và cực trị

Cho $a,b,c$ là những số không âm thoả mãn đẳng thức $a+b+c=1$.Chứng minh rằng:

$$a^2+b^2+c^2+9abc \ge 2(ab+bc+ac).$$

Ta có: Bất đẳng thức cần chứng minh tương đương với:

$$(a+b+c)^2+9abc\geq 4(ab+bc+ca)$$

$$\Leftrightarrow 9abc\geq 4(ab+bc+ca)(a+b+c)-(a+b+c)^3(\text{Vì }a+b+c=1)$$

BĐT cuối đúng theo $Schur$ nên ta có đpcm.




#640420 $Min:$ $$\frac{a}{ab+1}+\fr...

Posted by tpdtthltvp on 15-06-2016 - 08:27 in Bất đẳng thức và cực trị

Cho các số dương $a,b,c$ thoả abc = 1. Tìm $Min$ của $$\frac{a}{ab+1}+\frac{b}{bc+1}+\frac{c}{ca+1}.$$

Đặt $a=\frac{x}{y},b=\frac{y}{z},c=\frac{z}{x}$. Khi đó:

$$\sum \frac{a}{ab+1}=\sum \frac{\frac{x}{y}}{\frac{x}{z}+1}=\sum \frac{xz}{xy+yz}\geq \frac{3}{2}(Nesbit)$$




#639969 Inequalities From 2016 Mathematical Olympiads

Posted by tpdtthltvp on 13-06-2016 - 06:15 in Bất đẳng thức - Cực trị

Bài 41 (PAMO). Cho ba số dương $x,y,z$ thỏa mãn điều kiện $xyz=1.$ Chứng minh rằng
$$\frac{1}{(x+1)^2+y^2+1} + \frac{1}{(y+1)^2+z^2+1} + \frac{1}{(z+1)^2+x^2+1} \leqslant {\frac{1}{2}}.$$
 

Bài 41 (PAMO):  Áp dụng $AM-GM:$

$$\sum \frac{1}{(x+1)^2+y^2+1}=\sum \frac{1}{x^2+2x+1+y^2+1}\leq \frac{1}{2}\sum \frac{1}{xy+x+1}=\frac{1}{2}(\text{do }xyz=1)$$

Đẳng thức xảy ra khi $x=y=z=1.$

 

Bài 42 (EMMO). Cho $n$ số thực dương  $a_1,\,a_2,\ldots,\,a_n.$ Chứng minh rằng $$\prod_{i=1}^{n} \left(1+\frac{1}{a_i}\right)^{a_{i+1}-a_i} \geqslant 1,$$
trong đó $a_{n+1}=a_1.$

Bài 42 (EMMO): 

Áp dụng BĐT $Bernoulli,$ ta có:

$$\left(1+\frac{1}{a_i}\right)^{a_{i+1}-a_i}\geq 1+\frac{a_{i+1}-a_i}{a_i}=\frac{a_{i+1}}{a_i}$$

Do đó:

$$\prod_{i=1}^{n} \left(1+\frac{1}{a_i}\right)^{a_{i+1}-a_i}\geq \frac{a_2}{a_1}.\frac{a_3}{a_2}\cdots \frac{a_1}{a_n}=1$$




#639664 Đề Toán vòng 2 - tuyển sinh 10 chuyên Bình Thuận 2016-2017

Posted by tpdtthltvp on 11-06-2016 - 21:04 in Tài liệu - Đề thi

Đề Toán vòng 2 - tuyển sinh 10 chuyên Bình Thuận 2016-2017

Bài 3: Cho các số dương $x,y,z$. Chứng minh rằng:

$$\sum \frac{xy}{x^2+yz+zx}\leq \frac{x^2+y^2+z^2}{xy+yz+zx}$$

Bài 3: Tư tưởng chung là dùng BĐT $Cauchy-schwarz:$

$$(x^2+yz+zx)(y^2+yz+zx)\geq (xy+yz+zx)^2\Leftrightarrow \frac{xy}{x^2+yz+zx}\leq \frac{xy^3+xy^2z+z^2yz}{(xy+yz+zx)^2}$$

Thiết lập các BĐT tương tự, ta có:

$$\sum \frac{xy}{x^2+yz+zx}\leq \sum \frac{xy^3+xy^2z+x^2yz}{(xy+yz+zx)^2}=\frac{(xy+yz+zx)(x^2+y^2+z^2)}{(xy+yz+zx)^2}=\frac{x^2+y^2+z^2}{xy+yz+zx}(\text{đpcm}).$$




#639652 Tìm MIN P = $a^3+b^3+c^3$

Posted by tpdtthltvp on 11-06-2016 - 20:33 in Bất đẳng thức và cực trị

Cho a,b,c $\geq \frac{-1}{2}$ thỏa mãn $a^2+b^2+c^2=3$.

Tìm MIN P = $a^3+b^3+c^3$ 

Với $\forall x\geq -\frac{1}{2}$, ta có BĐT sau:

$$(x+\frac{1}{2})(x-1)^2\geq 0\Leftrightarrow x^3-\frac{3}{2}x^2+\frac{1}{2}\geq 0$$

$$\Leftrightarrow x^3\geq \frac{3}{2}x^2-\frac{1}{2}$$

Do đó ta có:

$$a^3+b^3+c^3\geq \frac{3}{2}(a^2+b^2+c^2)-\frac{3}{2}=3.$$

Vậy $\min P=3\Leftrightarrow a=b=c=1$




#639516 Inequalities From 2016 Mathematical Olympiads

Posted by tpdtthltvp on 11-06-2016 - 07:52 in Bất đẳng thức - Cực trị

Bài 40 (Syrian MO). Cho ba số thực dương $a,\,b,\,c$ thỏa mãn điều kiện $a+b+c=3.$ Chứng minh rằng
\[\frac{a^3+b^3}{a^2+ab+b^2}+\frac{b^3+c^3}{b^2+bc+c^2}+\frac{c^3+a^3}{c^2+ca+a^2} \geqslant 2.\]
Đẳng thức xảy ra khi nào ?
 

Bài nãy cũng quen thuộc :) :

Ta có:

$$\frac{a^2-ab+b^2}{a^2+ab+b^2}\geq \frac{1}{3}\Leftrightarrow \frac{2(a-b)^2}{3(a^2+ab+b^2)}\geq 0,\text{đúng}$$

Do đó:

$$\sum \frac{a^3+b^3}{a^2+ab+b^2}=\sum (a+b).\frac{a^2-ab+b^2}{a^2+ab+b^2}\geq \sum \frac{1}{3}(a+b)=2$$

Đẳng thức xảy ra khi $a=b=c=1$




#638908 4 - BĐT: Hàm số một biến

Posted by tpdtthltvp on 08-06-2016 - 13:00 in Bất đẳng thức - Cực trị

 

 Bài toán 6:

 

 

 Cho x, y và z là ba số thực dương.

 
 Hãy tìm giá trị nhỏ nhất của biểu thức:
 
$P=\frac{1}{3x+4y+4\sqrt{zx}}+\frac{1}{3x+2y+6\sqrt[3]{xyz}}-\frac{1}{\sqrt{7(x+y+z)}}.$
 
 
 P/S:
 
 Có bạn nào có hướng tiếp cận cho 2 Bài toán 2 và 5 chưa?

 

Áp dụng BĐT $AM-GM,$ ta có:

$$4\sqrt{xz}\leq x+4z$$

Và:

$$6\sqrt[3]{xyz}\leq x+2y+4z$$

Do đó:

$$P\geq \frac{1}{4(x+y+z)}+\frac{1}{4(x+y+z)}-\frac{1}{\sqrt{7(x+y+z)}}$$

Đặt $\sqrt{x+y+z}=t.$ Tới đấy xét hàm:

$$f(t)=\frac{1}{2t^2}-\frac{1}{\sqrt{7}t}$$